Where is the conclusion
So does the conclusion and the main point have different meanings or do they both mean the same t...
Lisam on September 6, 2022
  • September 2007 LSAT
  • SEC1
  • Q1
1
Reply
Now these questions, my mind turns into mash po...
I really need some elaboration on how to differentiate these "most strongly supported" questions ...
Tyler808 on August 5, 2022
  • September 2007 LSAT
  • SEC1
  • Q7
1
Reply
Whats the difference between C and E?
Help
Tyler808 on August 5, 2022
  • September 2007 LSAT
  • SEC1
  • Q6
1
Reply
Why doesn’t E weaken?
I debated this answer because it seems to provide an alternative cause of the observed effect
dannyod on July 22, 2021
  • September 2007 LSAT
  • SEC1
  • Q21
2
Replies
WHY E
Can someone explain how the answer is E and not C
saharK on January 15, 2021
  • September 2007 LSAT
  • SEC1
  • Q13
1
Reply
Biconditional
Would the second statement and first statement be a biconditional? If not, then how is it differe...
Timdec on January 13, 2021
  • September 2007 LSAT
  • SEC1
  • Q3
1
Reply
Please help
Please help explain why B is the correct answer and the others are not.
maonuo on October 8, 2020
  • September 2007 LSAT
  • SEC1
  • Q5
1
Reply
A & B
It is A because they talk about cause & effect? because current flow is "affected" by inductanc...
aseikhon11 on October 6, 2020
  • September 2007 LSAT
  • SEC1
  • Q7
1
Reply
Please explain.
Hello, Please explain, I chose B...
CMarr on September 14, 2020
  • September 2007 LSAT
  • SEC1
  • Q20
1
Reply
Please explain the question
I went with E... not sure for the reasoning on D
CMarr on September 14, 2020
  • September 2007 LSAT
  • SEC1
  • Q23
1
Reply
Why not B?
Please explain why not B..
CMarr on September 14, 2020
  • September 2007 LSAT
  • SEC1
  • Q25
1
Reply
Explanation
Can someone please explain the right answer?
zia305 on August 25, 2020
  • September 2007 LSAT
  • SEC1
  • Q22
3
Replies
Why not C?
Why is C incorrect?
kbernard on July 16, 2020
  • September 2007 LSAT
  • SEC1
  • Q24
3
Replies
Answer choice A for Q12
Hi I’m confused about why A is wrong? It seems decrease the credibility of non-worker owned busin...
Mnv on June 15, 2020
  • September 2007 LSAT
  • SEC1
  • Q12
1
Reply
What distinguishes B and E?
What is the logic that distinguishes these two?
bingolawyer on December 30, 2019
  • September 2007 LSAT
  • SEC1
  • Q24
1
Reply
Help
I do not understand the answer?
JayDee8732 on November 21, 2019
  • September 2007 LSAT
  • SEC1
  • Q24
4
Replies
Answer Explanation
Hi LSAT Max, (A) looked tempting at first. Could you explain why it would be flawed? Than...
Julie-V on September 11, 2019
  • September 2007 LSAT
  • SEC1
  • Q15
1
Reply
Answer Explanation
Hi LSAT Max, I was able to choose (A) but wasn't 100% confident about it. Could you help expl...
Julie-V on September 11, 2019
  • September 2007 LSAT
  • SEC1
  • Q10
1
Reply
Answer Explanation
Hi LSAT Max, Originally I chose (B) as my answer, but looking back I was wondering if the fla...
Julie-V on September 11, 2019
  • September 2007 LSAT
  • SEC1
  • Q2
1
Reply
Question
I was confused between a and d can you please explain?
kyoon on September 6, 2017
  • September 2007 LSAT
  • SEC1
  • Q3
1
Reply